subject
Mathematics, 31.07.2019 22:00 deanlmartin

Find the smallest possible set (i. e., the set with the least number of elements) that contains the given sets as subsets. {jill, john, jack}, {susan, sharon}

ansver
Answers: 1

Another question on Mathematics

question
Mathematics, 21.06.2019 17:00
For the rule of 78, for a 12-month period, the last term in the sequence is 12 and the series sums to 78. for an 10 month period, the last term is and the series sum is . for a 15 month period, the last term is and the series sum is . for a 20 month period, the last term is and the series sum is
Answers: 2
question
Mathematics, 21.06.2019 19:30
The wheels of a bicycle have a diameter of 70cm. how many meters will the bicycle travel if each wheel makes 50 revolutions
Answers: 2
question
Mathematics, 21.06.2019 21:30
Mis the midpoint of rq if rm equals 2x + 5 and mq equals 4x - 9 what is the length of rq
Answers: 1
question
Mathematics, 21.06.2019 21:40
Which statement is true about a number and its additive inverse? a. their product is always one. b. their sum is always one. c. they are always reciprocals of each other. d. their sum is always zero.
Answers: 1
You know the right answer?
Find the smallest possible set (i. e., the set with the least number of elements) that contains the...
Questions
question
Social Studies, 05.03.2021 22:00
question
Mathematics, 05.03.2021 22:00
question
English, 05.03.2021 22:00
question
Mathematics, 05.03.2021 22:00
question
Mathematics, 05.03.2021 22:00
question
History, 05.03.2021 22:00
Questions on the website: 13722367